Jump to content

Math04's Content

There have been 119 items by Math04 (Search limited from 06-06-2020)



Sort by                Order  

#736507 Chứng minh $m \leq 159$.

Posted by Math04 on 30-12-2022 - 10:55 in Tổ hợp và rời rạc

Trên mặt phẳng cho tập hợp $A$ gồm $66$ điểm phân biệt và tập hợp $B$ gồm $16$ đường thẳng phân biệt. Gọi $m$ là số bộ $(a,b)$ sao cho $a \in A, b \in B, a \in b$. Chứng minh $m \leq 159$.




#736506 Chứng minh rằng $n=2^k, k \geq 1$ hoặc $n=3.2^k, k \...

Posted by Math04 on 30-12-2022 - 10:50 in Số học

Cho $n \geq 2$ là số nguyên sao cho tồn tại $n-1$ số nguyên $x_{1},...x_{n-1}$ thỏa mãn: nếu  $0<i, j<n, i \neq j$ và $n$ là ước của $2i+ j$ thì $x_{i} < x_{j}$. Chứng minh rằng $n=2^k, k \geq 1$ hoặc $n=3.2^k, k \geq 0$




#736505 Tìm số nguyên dương $m$ nhỏ nhất sao cho: $2f_{1}(m)...

Posted by Math04 on 30-12-2022 - 10:44 in Số học

Với mỗi số nguyên dương $n$, gọi $D_{n}$ là tập hợp các ước dương của $n$ và $f_{i}(n)$ là số phần tử của tập hợp: $F_{i}(n)=\left \{a \in D_{ n}| a\equiv i (\text{mod} 4)  \right \}, i=1,2$. Tìm số nguyên dương $m$ nhỏ nhất sao cho: $2f_{1}(m)-f_{2}(m)=2017$.




#736504 Chứng minh tồn tại số nguyên dương $M$ sao cho: $a_{m...

Posted by Math04 on 30-12-2022 - 10:35 in Số học

Xét dãy $(a_{n})$ gồm vô hạn các số nguyên dương. Gỉa sử tồn tại số nguyên dương $N>1$ sao cho với mỗi $n\geq N$ thì $\frac{a_{1}}{a_{2}}+\frac{a_{2}}{a_{3}}+...+\frac{a_{n-1}}{a_{n}}+\frac{a_{n}}{a_{1}}$ là số nguyên. Chứng minh tồn tại số nguyên dương $M$ sao cho: $a_{m} = a_{m+1}, \forall m \geq M$.




#736345 Chứng minh dãy đã cho là dãy nguyên

Posted by Math04 on 18-12-2022 - 21:58 in Số học

Cho dãy: $\left\{\begin{matrix} u_{1}=1 & & \\ u_{n+1}=(1+\frac{3}{n})u_n+2-\frac{3}{n} & & \end{matrix}\right.$ Chứng minh dãy đã cho là dãy nguyên.




#736250 $f(x+y)+f(xy)=f(x)+f(y), \forall x,y \in \mathbb{R...

Posted by Math04 on 15-12-2022 - 11:26 in Phương trình hàm

Tìm $f:\mathbb{R}\rightarrow \mathbb{R}$ liên tục thỏa:

$f(x+y)+f(xy)=f(x)+f(y), \forall x,y \in \mathbb{R}$




#736249 $2f(x+y)+f(2x-2y)=f(2x)+f(2y)+2f(x-y), \forall x,y \in \m...

Posted by Math04 on 15-12-2022 - 11:25 in Phương trình hàm

Tìm $f:\mathbb{R}\rightarrow \mathbb{R}$ liên tục thỏa:

$2f(x+y)+f(2x-2y)=f(2x)+f(2y)+2f(x-y), \forall x,y \in \mathbb{R}$




#736248 Chứng minh các đường thẳng qua $X,Y,Z$ vuông góc $BC,CA,AB...

Posted by Math04 on 15-12-2022 - 11:22 in Hình học

Cho tam giác $ABC$ nhọn, không cân nội tiếp $(O)$. Ba đường cao $AH, BE, CF$. Trung điểm $AH, BH, CH$ lần lượt là $M,N,P$. $OM$ giao $BC$ tại $X$. Định nghĩa tương tự $Y,Z$. Chứng minh các đường thẳng qua $X,Y,Z$ vuông góc $BC,CA,AB$ đồng quy. 




#736246 Chứng minh rằng tồn tại số nguyên $a$, $1<a<\frac...

Posted by Math04 on 15-12-2022 - 11:18 in Số học

Bài này nhìn tưởng đơn giản nhưng để thỏa mãn điều kiện bất đẳng thức thì rắc rối ghê  :wacko:

 

Đặt phân tích ra thừa số nguyên tố của $n$ là $p_1^{\alpha_1}p_2^{\alpha_2}\dots p_k^{\alpha_k}$. Theo định lí thặng dư Trung Hoa, với mỗi $i\in \{1,2,\dots,k\}$ thì tồn tại số nguyên $x_i$ thỏa mãn

\[\left\{\begin{array}{l}x_i\equiv 1\pmod{p_i^{\alpha_i}}\\ x_i\equiv 0\pmod{\frac{n}{p_i^{\alpha_i}}}\end{array}\right..\]

Nhận xét (NX). Cho tổng $X=\sum_{i=1}^k\epsilon_ix_i$, trong đó $\epsilon_i\in \{0,1\}$ với mọi $i$.

  1. $n\mid X^2-X$.
  2. Nếu $X$ chia $n$ dư $0$ hoặc $1$ thì $\epsilon_1=\epsilon_2=\dots=\epsilon_k$.

 

Với mỗi $i\in \{1,2,\dots,k\}$, gọi $X_i$ là số tự nhiên nhỏ nhất thỏa mãn

\[X_i\equiv x_1+x_2+\dots+x_i\pmod{n}.\]

Theo NX2 thì $X_i\neq 0$ với mọi $i=\overline{1,k}$ và ngoài ra $X_k=1$. Bổ sung thêm $X_0=0$. Xét $k+1$ số $X_0,\ X_1,\ \dots,\ X_k$, theo định lí Dirichlet sẽ tồn tại $0\le c<d\le k$ sao cho

\[X_c,\ X_d\in\left [ \frac{jn}{k},\frac{(j+1)n}{k} \right ).\]

Nghĩa là $|X_d-X_c|<\frac{n}{k}$, theo NX2 ta có $X_d-X_c\notin \{0,1\}$. Cuối cùng ta chia ra hai trường hợp sau

$\bullet$ TH1: $X_d-X_c>1$ thì đặt

$$a_1=x_{c+1}+x_{c+2}+\dots+x_d=X_d-X_c\in\left(1,\frac{n}{k}\right).$$

Ngoài ra theo NX1 thì $n\mid a_1^2-a_1$ nên $a_1$ thỏa đề.

$\bullet$ TH2: $X_d-X_c<0$ thì đặt

$$a_2=(x_1+x_2+\dots+x_c)+(x_{d+1}+x_{d+2}+\dots+x_k)=X_k-(X_d-X_c)\in\left(1,1+\frac{n}{k}\right).$$

Ngoài ra theo NX1 thì $n\mid a_2^2-a_2$ nên $a_2$ thỏa đề.

Bạn có thể chia sẻ quá trình nghĩ ra lời giải này không nhỉ? Cám ơn bạn nhé!




#735852 Tìm min $M$

Posted by Math04 on 23-11-2022 - 22:42 in Bất đẳng thức - Cực trị

Cho $S>4$ và $P>S-3$ là các số cố định, Với bộ bốn số thực $(a,b,c,d)$ không nhỏ hơn $1$ sao cho $a+b+c+d=S$ và $abcd=P$, đặt $M=M(a,b,c,d)=max\left \{ a,b,c,d \right \}$. Tìm min $M$.




#735851 $|(a_{1}+...+a_{k})-(a_{k+1}+...+a_{n...

Posted by Math04 on 23-11-2022 - 22:37 in Bất đẳng thức - Cực trị

Cho số nguyên dương $n\geq 2$ và $n$ số thực $a_{1},a_{2},...,a_{n}$. Chứng minh tồn tại $k \in \left \{ 1,2,...,n \right \}$ sao cho:

$|(a_{1}+...+a_{k})-(a_{k+1}+...+a_{n})| \leq \underset{1 \leq i \leq n}{max}|a_{i}|$.




#735850 Tìm lim$(na_{n})$

Posted by Math04 on 23-11-2022 - 22:30 in Dãy số - Giới hạn

Cho dãy các số thực dương $(a_{n})$ thỏa hai tính chất: $\left\{\begin{matrix} a_{n+1}\leq a_{n}+a_{n}^2, \forall n \geq1 & & \\ a_{1}+a_{2}+...+a_{n} < M,\forall n \geq1 & & \end{matrix}\right.$ ($M$ là hằng số dương). Tìm lim$(na_{n})$.




#735849 Xác định số nghiệm âm và số nghiệm dương theo $n$ của $P_...

Posted by Math04 on 23-11-2022 - 22:26 in Đa thức

Cho dãy đa thức: $\left\{\begin{matrix} P_{0}(x)=0,P_{1}(x)=x & & \\ P_{n}(x)=(P_{n-1}(x))^3+2P_{n-1}(x)-P_{n-2}(x)+2022 & & \end{matrix}\right.$ Xác định số nghiệm âm và số nghiệm dương theo $n$ của $P_{n}(x)$.




#735790 Tồn tại đa thức bậc $ n$ có hệ số nguyên $p ( x )$ sao ch...

Posted by Math04 on 20-11-2022 - 15:46 in Đa thức

Chứng minh rằng với mỗi số nguyên dương $n$ ,tồn tại đa thức bậc $ n$ có hệ số nguyên $p ( x )$ sao cho $p ( 0 ) , p ( 1 ) , . . . , p ( n )$  là các số nguyên dương đôi một khác nhau,và tất cả chúng đều có dạng $2a^k + 3$




#735673 Chứng minh có thể chọn ra một tập con gồm $45$ điểm sao cho trong đ...

Posted by Math04 on 12-11-2022 - 19:04 in Tổ hợp và rời rạc

Trong mặt phẳng, cho $2023$ điểm sao cho không có $3$ điểm nào thẳng hàng:

a) Có ít nhất bao nhiêu tam giác không cân được tạo thành.

b) Chứng minh có thể chọn ra một tập con gồm $45$ điểm sao cho trong đó không có $3$ điểm nào tạo thành một tam giác đều.




#735616 Chứng minh rằng $(PQJ)$ tiếp xúc với $(O)$

Posted by Math04 on 06-11-2022 - 20:46 in Hình học

Cho tam giác $ABC$ nội tiếp đường tròn $(O)$. Tiếp tuyến tại $B,C$ của $(O)$ cắt nhau tại $P$ . $PO$ cắt $AC,AB$ tại $X,Y$ . Gọi $Q$ là trung điểm $XY$ . Qua $Q$ kẻ đường thẳng song song với $AO$ cắt
đường thẳng qua $P$ song song với $BC$ tại $J$.
a) Chứng minh rằng đường tròn đường kính $XY$ trực giao với $(O)$ .
b) Chứng minh rằng $(PQJ)$ tiếp xúc với $(O)$ .



#735614 Chứng minh $PH$ tiếp xúc với đường tròn $(GHL)$

Posted by Math04 on 06-11-2022 - 19:38 in Hình học

Cho tam giác $ABC$ nội tiếp đường tròn $(O)$. Đường tròn $(K)$ bất kì đi qua $B,C$ và cắt lại $AC, AB$ theo thứ tự tại $E,F$ . $BE$ cắt $CF$ tại $H$ . Trên $(O)$ lấy $G$ sao cho $\widehat{AGH}=90^{\circ}$ .$(BGH)$ cắt $(K)$ tại $P$ khác $B$, $L$ là điểm đối xứng của $H$ qua $B$. Chứng minh $PH$ tiếp xúc với đường tròn $(GHL)$.




#735530 Chứng minh rằng $X, Y, Z$ thẳng hàng

Posted by Math04 on 30-10-2022 - 22:39 in Hình học

Cho tam giác $ABC$ và điểm $M$ nằm trong tam giác. $AM, BM, CM$ lần lượt cắt $BC, CA, AB$ tại $D, E, F$. Lấy $X$ thuộc $BC$ sao cho $AM$ vuông $MX$ tại $M$. $Y,Z$ theo thứ tự là điểm đối xứng của $M$ qua $DE, DF$. Chứng minh rằng $X, Y, Z$ thẳng hàng.



#735483 Chứng minh rằng tồn tại số nguyên $a$, $1<a<\frac...

Posted by Math04 on 26-10-2022 - 22:02 in Số học

Cho $n$ là số nguyên lớn hơn $1$, $k$ là số các ước nguyên tố phân biệt của $n$. Chứng minh rằng tồn tại số nguyên $a$, $1<a<\frac{n}{k}+1$ sao cho $n|(a^2-a)$.



#735393 Tìm số các dãy $a_{1},a_{2},a_{3},...,a_{n}$ thỏa điều kiện đã cho...

Posted by Math04 on 20-10-2022 - 21:32 in Tổ hợp và rời rạc

Cho dãy $a_{1},a_{2},a_{3},...,a_{n}$ sao cho $a_{i} \in \left \{ 0,1,2 \right \}$ và $\left | a_{i}-a_{i+1} \right |\leq 1, \forall i$. Tìm số các dãy $a_{1},a_{2},a_{3},...,a_{n}$ thỏa điều kiện đã cho.




#735392 Tìm số các hoán vị của $n$ phần tử trong $B$ sao cho khôn...

Posted by Math04 on 20-10-2022 - 21:28 in Tổ hợp và rời rạc

Đặt $A=\left \{ 1,2,3,4 \right \}, B= \left \{ 1,2,3,...,n \right \}$. Tìm số các hoán vị của $n$ phần tử trong $B$ sao cho không có ba số hạng liên tiếp của nó mà mỗi số hạng đều nằm trong tập hợp $A$.




#735391 Tìm số các từ có độ dài $n$ trên một từ điển có 3 chữ cái $...

Posted by Math04 on 20-10-2022 - 21:23 in Tổ hợp và rời rạc

Với $n$ là số nguyên dương cho trước, tìm số các từ có độ dài $n$ trên một từ điển có 3 chữ cái ${a,b,c}$ sao cho chữ cái $a$ xuất hiện một số lẻ lần.




#735258 Chứng minh góc $QIO$ vuông

Posted by Math04 on 07-10-2022 - 23:02 in Hình học

Tam giác $ABC$ nhọn không cân nội tiếp $(O)$ với $I$ là tâm nội tiếp và $BE, CF$ là hai phân giác trong. $BE, CF$ cắt đường tròn $(O)$ tại $K, L$. $AI$ cắt $KL$ ở $P$ và $Q$ trên $EF$ sao cho $QP= QI$ . Chứng minh góc $QIO$ vuông.




#735257 Chứng minh $PM, QN, AX$ đồng quy

Posted by Math04 on 07-10-2022 - 22:59 in Hình học

Tam giác $ABC$ có $AD, BE, CF$ là các đường cao đồng quy ở $H$ và $X$ là tâm $(DEF)$, gọi $K, L$ là trung điểm $HB, HC$. Giả sử $FK$ cắt $DE$ tại $M, EL$ cắt $DF$ tại $N$ và $DK, DL$ lần lượt cắt $EF$ ở $P, Q$. Chứng minh $PM, QN, AX$ đồng quy.




#735245 Chứng minh dãy $u_{n}=\frac{n+1}{2^{n+1}}(\frac{2}{1}+\fr...

Posted by Math04 on 05-10-2022 - 22:45 in Dãy số - Giới hạn

Cho dãy: $u_{n}=\frac{n+1}{2^{n+1}}(\frac{2}{1}+\frac{2^2}{2}+...+\frac{2^n}{n}), n=1,2,3...$. Chứng minh dãy trên hội tụ và tìm giới hạn đó.